From: W. Trevor King Date: Fri, 20 Apr 2012 21:08:29 +0000 (-0400) Subject: Add question text and figures for Serway and Jewett v8's 25.7, .12, and .16. X-Git-Url: http://git.tremily.us/?p=course.git;a=commitdiff_plain;h=e60f485c89e50d178458a69ad9e66030fe3658e1;ds=sidebyside Add question text and figures for Serway and Jewett v8's 25.7, .12, and .16. --- diff --git a/latex/problems/Serway_and_Jewett_8/problem25.07.tex b/latex/problems/Serway_and_Jewett_8/problem25.07.tex index 083adda..238876f 100644 --- a/latex/problems/Serway_and_Jewett_8/problem25.07.tex +++ b/latex/problems/Serway_and_Jewett_8/problem25.07.tex @@ -1,4 +1,37 @@ \begin{problem*}{25.7} +A particle having charge $q=+2.00\U{$\mu$C}$ and mass $m=0.0100\U{kg}$ +is connected to a string that is $L=1.50\U{m}$ long and tied to the +pivot point $P$ in Figure~P25.7. The particle, string, and the pivot +point all lie on a frictionless, horizontal table. The particle is +released from rest when the string makes an angle $\theta=60.0\dg$ +with a uniform electric field of magnitude $E=300\U{V/m}$. Determine +the speed of the particle when the string is parallel to the electric +field. +\begin{center} +\begin{asy} +import Mechanics; +import ElectroMag; + +real u = 3cm; +real theta = 60; +pair a = u*dir(theta); +pair b = (u, 0); + +Angle t = Angle(a, (0,0), b, "$\theta$"); t.draw(); +draw(a -- (0,0) -- b); +dot("$P$", (0,0), align=W); +Charge A = pCharge(a, Label("$m$", align=N)); A.draw(); +A.lc.draw_label(Label("$q$", align=E)); +A.lc.draw_label(Label("$v=0$", align=W)); +label("$L$", a/2, align=NW); + +Vector v = Velocity(b, mag=u/3, dir=-90, "$\vect{v}$"); v.draw(); +Charge B = pCharge(b); B.draw(); +Vector E = EField((0.7u,u/2), mag=u/3, "$\vect{E}$"); E.draw(); + +label("Top view", (u/2, 0), align=S); +\end{asy} +\end{center} \end{problem*} \begin{solution} diff --git a/latex/problems/Serway_and_Jewett_8/problem25.12.tex b/latex/problems/Serway_and_Jewett_8/problem25.12.tex index 46f1c3c..b3a8dc2 100644 --- a/latex/problems/Serway_and_Jewett_8/problem25.12.tex +++ b/latex/problems/Serway_and_Jewett_8/problem25.12.tex @@ -1,4 +1,25 @@ \begin{problem*}{25.12} +The two charges in Figure~P25.12 are separated by $d=2.00\U{cm}$. +Find the electic potential at \Part{a} point $A$ and \Part{b} point +$B$, which is halfway between the charges. +\begin{center} +\begin{asy} +import Mechanics; +import ElectroMag; + +real u = 3cm; + +pair A = (u/2,sqrt(3)/2*u); +draw((0,0) -- A -- (u,0) -- cycle); +dot("$A$", A, align=N); +dot("$B$", (u/2,0), align=N); +Charge a = nCharge((0,0), Label("$-15.0\U{nC}$", align=S)); a.draw(); +Charge b = pCharge((u,0), Label("$27.0\U{nC}$", align=S)); b.draw(); +label("$d$", A/2, align=NW); +label("$d$", A/2 + (u/2,0), align=NE); +label("$d$", (u/2, 0), align=S); +\end{asy} +\end{center} \end{problem*} \begin{solution} diff --git a/latex/problems/Serway_and_Jewett_8/problem25.16.tex b/latex/problems/Serway_and_Jewett_8/problem25.16.tex index d7d0202..bdf7ea8 100644 --- a/latex/problems/Serway_and_Jewett_8/problem25.16.tex +++ b/latex/problems/Serway_and_Jewett_8/problem25.16.tex @@ -1,4 +1,24 @@ \begin{problem*}{25.16} +The two charges in Figure~P25.16 are separated by a distance +$d=2.00\U{cm}$, and $Q=+5.00\U{nC}$. Find \Part{a} the electric +potential at $A$, \Part{b} the electric potential at $B$, and \Part{c} +the electric potential difference between $B$ and $A$. +\begin{center} +\begin{asy} +import Mechanics; +import ElectroMag; + +real u = 2cm; + +draw(scale(u)*unitsquare, dashed); +dot("$A$", (0,u), align=N); +dot("$B$", (u,u), align=N); +Charge a = pCharge((u,0), Label("$2Q$", align=S)); a.draw(); +Charge b = pCharge((0,0), Label("$Q$", align=S)); b.draw(); +label("$d$", (0, u/2), align=W); +label("$d$", (u/2, 0), align=S); +\end{asy} +\end{center} \end{problem*} \begin{solution}